A class of irregular wavelet frames (Q1375820)

From MaRDI portal
scientific article
Language Label Description Also known as
English
A class of irregular wavelet frames
scientific article

    Statements

    A class of irregular wavelet frames (English)
    0 references
    0 references
    0 references
    0 references
    9 April 2001
    0 references
    Let \(f\in L^2(\mathbb{R})\), \(\widehat f(\omega)= \int_{\mathbb{R}} f(x)e^{-2\pi ix\omega}dx\) the Fourier transform of \(f\). For \(\Omega>0\), we write \[ B_\Omega= \{f\in L^2(\mathbb{R}): \text{supp }\widehat f\subset [-\Omega,\Omega]\},\quad L^2[-\Omega, \Omega]= \{f\in L^2(\mathbb{R}): \text{supp }f\subset [-\Omega,\Omega]\}. \] If \(\{g_k\}\) is a sequence of functions in \(L^2(\mathbb{R})\) such that there exist two positive numbers \(A\) and \(B\) satisfying \[ A\|f\|^2\leq \sum_k|\langle f,g_k\rangle|^2\leq B\|f\|^2,\quad \forall f\in L^2(\mathbb{R}), \] than \(\{g_k\}\) is called a frame. Suppose \(g\in L^2(\mathbb{R})\), \(\{s_k\}_{k\in \mathbb{Z}}\) be a sequence of positive and \(\{a_k\}_{k\in \mathbb{Z}}\) a sequence of real numbers. In this article the authors give some conditions, by which \(\{g_{j,k}\}_{(j,k)\in \mathbb{Z}^2}\) constructs a frame in \(L^2(\mathbb{R})\), where \(g_{j,k}(x)= {1\over\sqrt{s_j}} g({x\over s_j}+ a_k)\). Let \(e_\alpha(x)= e^{2\pi i\alpha x}\) and \(\text{fix }\Omega>0\) and let be given a sequence \(\{a_k\}_{k\in \mathbb{Z}}\) of real numbers. If for \(A\) and \(B>0\), \(\forall f\in L^2[-\Omega, \Omega]\) \[ A\|f\|^2\leq \sum_k|\langle f,e_{a_k}\rangle|^2\leq B\|f\|^2, \] then \(\{e_{a_k}\}_{k\in \mathbb{Z}}\) is called a Fourier frame in \(L^2[-\Omega, \Omega]\) with bounds \(A\) and \(B\). Main result of this article is Theorem 1: Assume that the following three conditions are satisfied: (i) \(g\in B_\Omega\cap L(\mathbb{R})\), (ii) \(a\leq \sum_j|\widehat g(s_j\omega)|^2\leq b\) a.e. \(\omega\), where \(\{s_j\}_{j\in \mathbb{Z}}\) is a sequence of positive numbers and \(a\), \(b\) are fixed positive number; (iii) \(\{e_{a_k}\}\) is a Fourier frame in \(L^2[-\Omega, \Omega]\) with bounds \(A\) and \(B\). Then \(\{g_{j,k}\}_{(j,k)\in \mathbb{Z}^2}\) is a frame in \(L^2(\mathbb{R})\) with bounds \(aA\) and \(aB\).
    0 references
    0 references
    irregular wavelet frames
    0 references
    Fourier frame
    0 references